Search found 19 matches


Probability Problem

Hi, please help me with this problem !!! Thanks in advance ...


THE OA is D


ANSWERS:

a. 1/10
b. 1/8
c. 1/6
d. 1/5
e. 1/4

by alsergi

Tue Nov 04, 2008 2:49 pm
Forum: Problem Solving
Topic: Probability Problem
Replies: 8
Views: 1839

GMATPrep Geometry Problem Lenght Arc Parallel lines

Hi! Please help me with this. I have searched for this question in the forum but havent found ....

Thanks in advance, the OA is A.

by alsergi

Tue Nov 04, 2008 12:52 am
Forum: Problem Solving
Topic: GMATPrep Geometry Problem Lenght Arc Parallel lines
Replies: 2
Views: 2610

stop@800 wrote:is OA C?
Sorry I forgot, Yes, the OA is C

by alsergi

Wed Oct 08, 2008 4:07 am
Forum: Problem Solving
Topic: Powers and integers
Replies: 4
Views: 1815

Powers and integers

Please, could you help me with this? Thanks in advance!!!

If the units digit of n^33 is 7, which of the following could be the value for n?

I. n=41
II. n=43
III. n=47

a. Only I
b. Only II
c. Only III
d. I and II
e. II and III

by alsergi

Wed Oct 08, 2008 1:24 am
Forum: Problem Solving
Topic: Powers and integers
Replies: 4
Views: 1815

Inequities

Hope you could help me with this:

Thank you all!!!


Is a > b^2 ?

1. a>b^4
c. a >sqrt (b)

The right answer is C. ¿?

by alsergi

Sun Sep 28, 2008 7:26 am
Forum: Data Sufficiency
Topic: Inequities
Replies: 5
Views: 1711

Re: Inequities Data Sufficiency

Hi All !!! Could anyone help me with this, please? Thanks in advance: Is x <= 3 ? 1. 6x<= 24 2. 3x+1>=12 The answer is supposed to be D, but I dont get the point, I should tried B ... I dont understand the D since 1 states that x <= 4, so it could be less than 3 or between 3 and four so I think is ...

by alsergi

Fri Sep 26, 2008 10:05 am
Forum: Data Sufficiency
Topic: Inequities Data Sufficiency
Replies: 2
Views: 1441

Inequities Data Sufficiency

Hi All !!! Could anyone help me with this, please? Thanks in advance: Is x <= 3 ? 1. 6x<= 24 2. 3x+1>=12 The answer is supposed to be D, but I dont get the point, I should tried B ... I dont understand the D since 1 states that x <= 4, so it could be less than 3 or between 3 and four so I think is n...

by alsergi

Fri Sep 26, 2008 8:06 am
Forum: Data Sufficiency
Topic: Inequities Data Sufficiency
Replies: 2
Views: 1441

Yes, the answer is D, thank you all !! You totally Rock :mrgreen:

by alsergi

Thu Sep 25, 2008 3:56 am
Forum: Data Sufficiency
Topic: remainder data sufficiency question
Replies: 8
Views: 2346

Re: Powers and roots

Yes, definitely the right answer is B. Thank you all !!!!

alsergi wrote:Please help me with this!!!! Thanks in advance.

If 2^6 < x < 2^8, is x closer to 2^6 or 2^8 ?

1. x is closer to 2^4 than to 2^9
2. x is closer to 2^5 than to 2^7

by alsergi

Thu Sep 25, 2008 3:48 am
Forum: Data Sufficiency
Topic: Powers and roots
Replies: 6
Views: 1645

Powers and roots

Please could anyone help me with this?
Thanks in advance ...

If ( 1/2 * (400^300 + 20^ 600) )^4 = 400^k what is the value of k ?

a. 400
b.600
c. 800
d. 1200
e. 2400

Right answer is d, but I dont know how to solve it ...

by alsergi

Thu Sep 25, 2008 3:13 am
Forum: Problem Solving
Topic: Powers and roots
Replies: 2
Views: 1848

Powers and roots

Please help me with this!!!! Thanks in advance.

If 2^6 < x < 2^8, is x closer to 2^6 or 2^8 ?

1. x is closer to 2^4 than to 2^9
2. x is closer to 2^5 than to 2^7

by alsergi

Wed Sep 24, 2008 1:30 am
Forum: Data Sufficiency
Topic: Powers and roots
Replies: 6
Views: 1645

remainder data sufficiency question

Please, could you help me with this question? If n is the sum of three consecutive nonnegative integers p, s and t, with p<s<t, and n is also the product of three consecutive nonnegative integers x, y and z, with x<y<z What is the remainder when n is divided by 5? 1. The remainder when p is divided ...

by alsergi

Tue Sep 23, 2008 3:48 am
Forum: Data Sufficiency
Topic: remainder data sufficiency question
Replies: 8
Views: 2346

Thank you very much sudhir3127, but I dont agree with you ... I agree with the first statement but not with the second 1 - 1/10 would be the probability of AT LEAST one man, not the probability of 2 women so, you cant answer B ... The right answer is E, but I dont understand exactly why ... Thank yo...

by alsergi

Fri Aug 22, 2008 5:11 am
Forum: Data Sufficiency
Topic: Probability Problem
Replies: 5
Views: 1695

Probability Problem

Please, help me with this, problem, I am a little bit confused ... If 2 different representatives are about to be selected from a group of 10 employees and if x is the probability that both representatives are women , is x > 1/2 ? 1. More than 1 / 2 of the employees are women. 2. The probability tha...

by alsergi

Fri Aug 22, 2008 1:46 am
Forum: Data Sufficiency
Topic: Probability Problem
Replies: 5
Views: 1695

Yes, I see your point, the result is B.

Thank you very much!!

by alsergi

Thu Aug 21, 2008 8:37 am
Forum: Data Sufficiency
Topic: Inequities
Replies: 3
Views: 1622